« first day (44 days earlier)      last day (530 days later) » 
00:00 - 16:0016:00 - 00:00

12:01 AM
0
Q: Tensor product for vector bundles is commutative, associative, and has an identity element?

Jason IsmailHow do I see that the tensor product operation for vector bundles over a fixed base space is commutative, associative, and has an identity element?

0
Q: Question about varying density of a sphere to find its mass.

Uys of SpadesI have a question about the process to find the mass of a sphere with a varying radial density in respect to the radius. It's something really simple, but I would like someone to explain it me. Say that the density varies with: $\rho(r)=a-br$ So when you go from $m=\rho V$ Why is it that it c...

0
Q: Convolution Problem

Nickwhile working on a signal processing problem i've reached to the following: So my aproach was: Am I doing something wrong? Is it valid Y(f)=[X(f) x H(f)]*W(f)=X(f) x [H(f)*W(f)] If you could help me move further simplyfing Y(f) Thank you in advance

Short title. Title contains problem. Convolution Problem
0
Q: Find distinct left cosets of SL(2,R) in GL(2,R)

ycex34Find distinct left cosets of SL(2,R) in GL(2,R) I am guessing that it should be for a in R, a*SL(2,R) Is this true?

0
Q: Combinatorial Probabilities

JessieI've been trying out some of the early problems in the probabilistic method, since I hear it can prove many problems in discrete mathematics. Let $A_1,..., A_n$ be events such that the independent of all events except for at most $m$ of them. Taking $0 < \epsilon < 1,$ and $Pr[A_i] \leq \frac{\ep...

0
Q: Prove that there is some points $x_0$ in the interval $[a,b]$ at which $f(x_0)=0$.

SimpleSuppose that the continuous function $f:[a,b]\rightarrow\mathbb{R}$ has $\int_{a}^{b}f(x)=0$. Prove that there is some points $x_0$ in the interval $[a,b]$ at which $f(x_0)=0$. Proof: Since $\int_{a}^{b}f(x)=0$, there exists a partition sequence $\{P_n\}$ such that $$\sup\{L(f,P_n)\}=\inf\{U(...

Tagged proof-verification. [Prove that there is some points $x_0$ in the interval $[a,b]$ at which $f(x_0)=0$.](math.stackexchange.com/q/1577637)
0
Q: Help proving a lemma

feralinI'm trying to prove the following lemma: We have $m$ is a real number greater than 1, and an open interval $(a, b) \subseteq [0, m)$ with $a < b$. Then if $n\text{ mod } m \in (a, b)$ for some integer $n$, then there is another integer $j > n$ so that $j\text{ mod }m$ is also in $(a, b)$. This ...

Words such as help do not add information to titles. Please edit the title so that it better describes the specifics of your question. Do not hesitate to make it longer or include a formula if needed. More tips here. (autocomment)Normal Human 21 secs ago
 
0
Q: What did I miss when suggesting an edit to a Windows Command Prompt question?

tepplesI tried to improve the question How to copy the contents in cmd prompt to notepad? in two ways: to improve its English language usage and to bring it on-topic by explicitly stating how it relates to programming tools. Yet three Stack Overflow users rejected it, commenting: This edit does not ...

0
Q: Answering a question with other problems

Robbie AverillThis question is similar to this one regarding SQL injection. Assuming somebody posts a question (like this one) saying "What's wrong with my query?" and the answer is syntax related, or something of the like, what's the best approach to answering? Answer the question, pointing out the syntax ...

0
Q: The live refresh randomly starts getting questions without the summary div... again

BraiamSeeing the same issue that The live refresh randomly starts getting questions without the summary div has, just that in that case it was fixed, but the problem reappeared. Is trivially reproducible, just open some active tag page and wait. Probably related to this specific change: the page...

 
12:17 AM
0
Q: Kernel of continuous operator on a Banach Space

Anthony PeterLet $z \in \mathcal B$ be non-zero. Let $T \in \mathcal B^*$ be defined as the mapping $$ \operatorname{span}\{z\} \ni \alpha z \mapsto \alpha \in \mathbb{C}$$ and $T(y-z) = 0$ for $y \notin \operatorname{span}\{z\}.$ Is it then true that $$\mathcal B = \operatorname{ker} T \oplus \operatorname...

 
12:34 AM
0
Q: Given that the mass of the earth is approximately 6 * 10^24 kg, the mass of the sun is about 2 * 10^30, the earth is about 1.5 * 10^10 m...

pidgey...How can I calculate the work required to move the Earth 1 cm farther away from the sun, given that the gravitational constant is 6.7 * 10^-11 m^3/kg sec^2?

0
Q: Show the following tableau is valid by tableau method

Chen Lu$a1. ∃x [p(x) ∧ ∀y [p(y) → q(x, y)]]$ given $a2. ¬ ∃x [p(x) ∧ q(x, x)]$ given

0
Q: Liouville's Theorem Derivative solving?

user299020I believe this is a Liouville's Theorem problem, but I am unsure as to how to solve it. |f(z)|< |z^2 + 3z +1| for all z, and that f(1) = 2. Evaluate f'(2), and explain your answer.

0
Q: Profit Maximization Problem

matherialsFind the maximum profit corresponding to a $\ demand \ function \ $ of $ \ p=36-4x$ and a total $cost\ function \ = 2x^2+6$ Hello. Can you kindly help me solve this problem? Thank you in advance

Short title. Title contains problem. Profit Maximization Problem
0
Q: How many combinations are possible?

EGMI have 12 objects of 3 different types: 5 type A, 4 type B, and 3 type C. These objects must be arranged into 6 pairs such that no type is paired with another of the same type. The only arrangement I can come up with is: Three A/B, Two A/C, One B/C There might not be another way; however, I'm ...

Consider adding a tag for a broader subject area to which the question belongs. Some of these tags might fit. (autocomment)Normal Human 21 secs ago
0
Q: Question related to decay of Fourier transform and smoothness

Johnny T.Suppose that $f\in L^1 (\mathbb{R})$ and that for any $n\in \mathbb{N}$ there is $C_n > 0$ such that its Fourier transform satisfies $$ |\hat{f}(\xi )| \le C_n(1+|\xi |^2)^{-n}. $$ Let $$g(x) = \frac 1{2\pi}\int_{\Bbb R}\widehat f(\xi)e^{i\xi x}d\xi$$ and suppose we want to show that $g$ is smo...

0
Q: Would this make math simpler? If not, what problems would it create?

ZduffSo I'm refreshing on integration and venture across a property of definite integrals I've been taking for granted for quite some time: \begin{equation}\int_{a}^{b} f(x) \mathrm{d}x =-\int_{b}^{a} f(x) \mathrm{d}x \end{equation} Stewart's Calculus claims this property is owed to $\Delta x= \dfra...

-1
Q: Stumped on a Smash Brothers Math Dillema

user299022If I have the numbers 1, 2, and 3 and there are 4 different places (Example: 1111 or 1211 or 2312), how many different combinations can you have for the numbers 1, 2, and 3?

Title contains stump. Short question. Stumped on a Smash Brothers Math Dillema
0
Q: Optimization problem involving rectangles [Calculus 1]

user299023Spent WAY too long trying to figure this out and I just don't know what I am doing wrong. "A rectangular region is to be fenced using 5100 feet of fencing. If the rectangular region is to be separated into 4 regions by running 3 lines of fence parallel to two opposite sides, determine the dimens...

Welcome to Math.SE, user299023. This site uses MathJax formatting of formulas. More tips here. (from a bot)Normal Human 21 secs ago
0
Q: What is this geometry question saying?

John RyanProblem: A few questions: what parallelogram is the solution talking about? Also how is its height $\dfrac{1}{2}$ and base $\dfrac{\sqrt{2}}{2(1+\sqrt{2}}$? Here is a picture of just the squares:

Words such as question do not add information to titles. Please edit the title so that it better describes the specifics of your question. Do not hesitate to make it longer or include a formula if needed. More tips here. (from a bot)Normal Human 21 secs ago
 
1:02 AM
0
Q: Clicking on "Edit CV" takes me to sign up

Travis JI opted in to see the new beta "Jobs" section as noted in the question Careers Unificintegration: Jobs on Stack Overflow. I do not have a Careers account. When I went to my profile, I noticed a new option "Edit CV". I clicked it out of curiosity and it took me to the sign in page for Stack Ov...

 
0
Q: Key Result in Group Theory

JoshI've been going about doing one of the key results in Group Theory: Given a group $G$, for all subgroups $H \leq G,$ $o(H)$ is a multiple of $o(G).$ I have a feeling a good start to this would be using the properties of $\mod H$ to start this, because it seems to me that $\mod H$ partitions $G$ i...

0
Q: Ellipse Contour Integral

user299020I'm having trouble with the following complex integral: g(w) = ∫ z exp(z^3)/(z-w)^3 dz where C is a positively oriented ellipse x^2/9 + y^2/4 = 1 Find g(2+3i) in the form a +bi I know z(t) = 3cos(t)+2isin(t) However, when I substitute this in (along with the z '(t)) the integral becomes a mess...

This site uses MathJax formatting of formulas. More tips here. (from a bot)Normal Human 21 secs ago
0
Q: Volume between (z = 3*sqrt(x^2 + y^2) and (x^2 + (y-1)^2 = 1) and (z = 0)

user2615936I am not sure how to approach finding the limits of integration. Would I need to change coordinate systems?

0
Q: How do I prove that a hypocycloid, which has equation x^(2/3) + y^(2/3) = a^(2/3), can be parameterized by x = acos^3(/Theta), y = asin^3(/Theta)?

cantthinkofanameThe problem assumes that it is true, but I'm not quite sure how to go about proving it. Where do I proceed?

 
1:29 AM
0
Q: Polynomial in $\mathbb F[x]$

OscarI'm brushing up on algebra, reading over some questions I found online. I was wondering, if $p(x)\in \mathbb F[x]$, for some field $\mathbb F$, then what is $\langle p(x)\rangle$? Does it mean the elements like $p(x)+p(x)$, or elements like $p(x)p(x)$? Or something else? Thanks!

Short title. [Polynomial in $\mathbb F[x]$](math.stackexchange.com/q/1577719)
 
1:40 AM
0
Q: Why is taking the average of the x values and y values of the endpoints of a segment will result in the coordinates of the midpoint?

user299030Why is taking the average of both the x and y values of the endpoints of a segment result in the coordinates of the midpoint?

0
Q: Proving reasoning by cases.

LagraMen0123I am trying to prove the following statement: Let $X, Y , Z$ be random variables, then $P(X \vert Y) = \sum_{z} P(X,z \vert Y)$. I have a sketch of the proof but I do not know if it is correct: $$\sum_{z} P(X,z \vert Y) = \sum_{z} \frac{P(X,Y,z)}{P(Y)} =$$ $$\frac{\sum_{z}P(X,Y,z)}{P(Y)}...

0
Q: Lagrange Interpolation With Expand Returning a Wrong Result in Maxima

Nathan CamposRecently I've been experimenting with polynomial interpolation to create a simple model of a battery discharge curve. My mathematics knowledge is a bit limited and this is part of my quest to find interesting things that my professors never mentioned. I've found something very interesting while ...

Welcome to Math.SE, Nathan Campos. Consider adding a tag for a broader subject area to which the question belongs. Some of these tags might fit. (autocomment)Normal Human 21 secs ago
0
Q: A different way to prove homeomorphism between rectangles and disc under standar topology

MasacrosoI was reading about the classical problem to prove homeomorphism between subspaces of $\Bbb R^2$ a rectangle of the kind $R=\{(x,y):|x|\le a \land |y|\le b\}$ and some disc $D=\{(x,y): x^2 + y^2 \le r\}$. It is clear that we can make bijections between the two spaces but the hard part is take so...

0
Q: On the little oh notation.

MonoliteI have been told that in a fraction if we only have little ohs then we can't conclude anything taking the limit. As an example say we have $$ \lim_{x \rightarrow 0}\frac{o(x^3)}{o(x^2)}$$ then we can't conclude anything but this confuses me slightly because it seems to me we should be able to sa...

0
Q: bound the norm of a L2 function

Joaquín MoragaLet $(X, \mu)& be a $\sigma $-finite measure space, $f $ be a $L^2 (X)$ function, $K (x,y)$ a $L^2 (X\times X)$ function. Define $Tf (x)=\int_X K (x,y)f (y)dy$. I need to prove that $||Tf||_2 \leq ||K||_2||f||_2$. I can bound the left side with $\int_X |f (y)|^2 \int_X |K (x,y)|^2 dx dy$. But I...

Consider replacing (analysis) with a more specific tag for the relevant branch of analysis. (from a bot)Normal Human 21 secs ago
0
Q: Non-empty collection of balls

user2820579The problem states: How many non-empty collection of balls can be formed from x-red balls, y-red-balls, and z-green balls. I don't want the answer, but rather the explanation of what is "non-empty collection". To me it sounds something like the elements in the power-set of colored balls minus t...

0
Q: Changing the measure

truebaranSuppose that I have the following situation: everything takes place on complex plane, I have some probabilistic measure $\mu$ and suppose that I have a family of function $(u_w)_w$ parametrised by $w$ where $w$ runs throught some subset of the complex plane (it may be whole $\mathbb{C}$). Suppose...

Short title. Changing the measure
0
Q: What is the value of $142,857 \times 7^2$?

John Ryan What is the value of $142,857 \times 7^2$? Obviously you could solve this with a calculator and be done. But is there a more clever way to calculate this?

 
2:23 AM
0
Q: A proof of $(\forall x P(x)) \to A) \Rightarrow \exists x (P(x) \to A)$

Michael WelchI recently asked this question. In that question I presented a hand-waving proof as part of the question. There was some confusion as to the validity of my hand-waving proof. So I wanted to make it more precise. The difficulty that I had is that it's been 25 years since I've written formal logic ...

 
0
Q: Is 'icons' a good tag?

Al E.I draw your attention to icons. On the (currently) 10 questions with that tag... 1 is about emoji in Gmail subject lines 2 are about emoticons in Facebook 3 are about icons in Trello 1 is a request for an icon generator 1 is about icons in Yahoo! Mail 1 is a request for an icon search service ...

 
2:40 AM
0
Q: Is providing an answer to comment area a good practice?

Tareq MahmoodThere are several times I saw people are giving solutions of the problem in the comment area. Some of them later provide an answer themselves, others can be converted to answers by making it "community wiki" as suggested by this post : Question with no answers, but issue solved in the comments. ...

0
Q: Synonymize [iso-639-2] and [iso-639]

ivan_pozdeevThere are several standards in ISO 639 family, but they all are pretty much about the same and are rather rare a topic. So I don't think each one warrants a separate tag. The specific incentive was Convert ISO 639-1 to ISO 639-2 that would otherwise need two tags, iso-639-2 and iso-639-3, the la...

 
0
Q: About −| x |≤ x ≤| x | in absolute values

mathnoobSo, i'm really strugling with this one, when studying the triangle inequality, the inequality $−|x|≤x≤|x|$ pops really often, however I just can't get why this is true. I've seen only two different approaches, one that it seems to me too trivial: Since: $|x|≤a⇔−a≤x≤a$ We can write: $|x|≤|x...

0
Q: Determine whether the function rule models discrete or continuos data

AbelA produce stand sells roasted peanuts for $2.99 per pound. The function C(p)=2.99p relates the total cost of the peanuts to the number of pounds purchased p

 
2:59 AM
0
Q: Unable to remove "top answer" from the list

Nathan OsmanI am trying to remove the following "top answer" from my CV: There are at least three problems here: the "14 votes" text is overlaying the up and down arrows the "remove" link is very difficult to click the "remove" link doesn't work When I click the link, I receive a confirm() dialog aski...

 
0
Q: characterization in l2 spaces

Jaynotcan anyone please give me some clarity on this question that if $||\frac{f_n}{x-i}||_{2}$ goes to 0 as n goes to infinity. how can i justify that $||f_n||_{2}$ goes to 0 also? I have been trying to use the Holders inequality but I am getting the whole thing messed up

Short title. Question contains please. characterization in l2 spaces
 
0
Q: I see my stackoverflow reputation score at the top bar instead of meta.stackexchange score when I switch from stackoverflow

Tareq MahmoodI suddenly faced the issue today. I was in stackoverflow and I switched to Meta StackExchange by using the navigation at the top left dropdown. I noticed that I am seeing the reputation score of my stackoverflow profile at the right side of the top bar. I tested it multiple times and it did the...

 
3:17 AM
0
Q: Rubik's Revenge Cube in GAP

I Can Haz HatsI'm trying to create the Rubik's Revenge (4x4x4 cube) group in GAP . Take the following net of the 4x4x4 cube with each sticker labelled with a number. The front, left, upper, right, down, and back faces labelled with their respective initials. U [64][65][6...

Questions tend to get more attention when they have a tag for a broad area of mathematics relevant to the question. Some of these tags might fit. (autocomment)Normal Human 20 secs ago
0
Q: Taylor Series Exercise

Marko PoloWhat method should I use to find the Taylor series of $f(x)=\frac{x+2}{2-3x}$ with center 2?

Short title. Title contains exercise. Short question. Taylor Series Exercise
0
Q: Solution of $\frac{dy}{dx}=\frac{1}{y}-\sqrt{4x-x^2}$?

lanse7ptySolution of $\frac{dy}{dx}=\frac{1}{y}-\sqrt{4x-x^2}$? I dont know how to get the solution of the equation.Thanks for any hint or detail answer.

Consider replacing (analysis) with a more specific tag for the relevant branch of analysis. (autocomment)Normal Human 21 secs ago
Consider replacing (analysis) with a more specific tag for the relevant branch of analysis. (from a bot)Normal Human 21 secs ago
0
Q: Proving a group has a subgroup of a certain order

user299046The question is to show if $G$ has order 77 then $G$ has a subgroup of order 7. Without using Sylow Theorems. Attempt sketch: Let $x \in G$. By Lagrange's theorem the order of $x$ is either $1, 7, 11$. Suppose $x \neq e$ Then $x$ has order $7$ or $11$. Now suppose $|x| = 7$. Then $x$ is the ge...

0
Q: Coupled, linear, non-constant coefficient second order differential equations

Graham ReidI have a question concerning an eigenvalue problem that I've been working on for a while now. Essentially, I have a system of coupled, linear, non-constant coefficient, second order differential equations of the following form: $\frac{\partial^2 y_1}{\partial x^2} + a_1(x) \frac{\partial y_1}{\p...

Tagged differential-equations but mentions "partial". Coupled, linear, non-constant coefficient second order differential equations
0
Q: statistics: exponential scale family

GRAAMLet X1, X2,....Xn be a random sample from the exponential distribution with the pdf f(x│θ)=(1/θ) e^((-x)/θ) where x≥0 find E(T_i ) (MEAN), i=1,2,….,n, where T_i= X_i /(X_1+X_2+⋯+X_n ) would you please help me to solve this problem

Welcome to Math.SE, GRAAM. This site uses MathJax formatting of formulas. More tips here. (autocomment)Normal Human 21 secs ago
 
3:46 AM
0
Q: I deleted my question and now I received a banned notification

Pablo TobarI did asked a question this afternoon about WCF and just at the time it was posted I received a negative vote and then all of a sudden I received another negative vote so my question didn't got any answers but it already had two negative votes so I decided to delete the question because in my opi...

 
0
Q: Is $T$ a bijection?

John DoeThe question is Let $T: \mathbb{R}^3\rightarrow \mathbb{R}^3$ be defined by $T(v) = Av$ wheere $A$ is given by $A = \frac 13 \begin{bmatrix} 2 & -1 & 2 \\ 2 & 2 & -1 \\ -1 & 2 & 2 \\ \end{bmatrix}$ $T$ is a bijection if it is both injective and surject...

Short title. Is $T$ a bijection?
0
Q: Help me find an explicit formula from a recursive formula

Billy DeeI need help finding an explicit formula from this recursive formula: a_n = a_n-1 -3/5(n-1) a_1 = 1/2

Welcome to Math.SE, Billy Dee. Words such as help are uninformative in titles. Please edit the title so that it better describes the specifics of your question. Do not hesitate to make it longer or include a formula if needed. This site uses MathJax formatting of formulas. More tips here. (autocomment)Normal Human 20 secs ago
0
Q: Primitive Root Modulo $p=4q+1$

MaburoLet $p$ and $q$ be primes such that $p=4q+1$. Then $2$ is a primitive root modulo $p$. Proof. Note that $q\not=2$ since $4\cdot2+1=9$ is not prime. $\mathrm{ord}_p(2)\vert p-1=4q$, so $\mathrm{ord}_p(2)=1,\;2,\;4,\;q,\;2q,\;\mathrm{or}\;4q$. Clearly $\mathrm{ord}_p(2) \not= 1$, and $\mathrm{o...

Tagged proof-verification. Primitive Root Modulo $p=4q+1$
0
Q: Solution of $7$-th order ODE

S.Panja-1729Let , $y=x\sin x+x^2$ be a solution of the $7$-th order ODE $$a_7y^{(7)}+a_6y^{(6)}+\cdots +a_0y=0.$$Then which of the following are correct ? (A) $a_7+a_3=a_5$. (B) $\displaystyle \sum_{i=0}^7 a_i=4$. (C) $\displaystyle \sum_{i=0}^7 a_i=3$. First I differentiate $y$ ,$7$-times and then put t...

Short title. Question contains please. Solution of $7$-th order ODE
0
Q: Rank of quadratic matrix form

user23658Let $p > r$. Suppose I have a $p \times r$ matrix $V$ of rank $r$ and a $p \times p$ positive semi-definite matrix $X$ of rank $q$. What does this imply about the rank of $$ V^T X V?$$ Is this matrix positive definite if $r < q$?

0
Q: Problem on normal and symmetric matrices

user299054A normal matrix over $\mathbb C$ with all eigenvalues real is hermitian (using diagonalization) .But a normal matrix with real eigenvalues is symmetric, is this statement true? I think, all normal matrices are not diagonalizable i.e rotation matrix .please someone explain over $\mathbb R$,are t...

Title contains problem. Question contains please. Problem on normal and symmetric matrices
0
Q: Let G=Z4+U(4), H=<([2]4,[3]4)>, K=<([2]4,[1]4)>. Show that G/H is not isomorphic to G/K.

ycex34Let G=Z4+U(4), H=<([2]4,[3]4)>, K=<([2]4,[1]4)>. Show that G/H is not isomorphic to G/K. [2] means 2 mod 4.

Short question. [Let G=Z4+U(4), H=<([2]4,[3]4)>, K=<([2]4,[1]4)>. Show that G/H is not isomorphic to G/K.](math.stackexchange.com/q/1577852)
0
Q: Question on combinatorics about pairing.

HumbleStudentIf 2n people are sitting at equally spaced intervals around a circle. How many ways can they form n pairs if no two persons seated directly opposite each other can form a pair? What I did was draw some examples for low values of 'n' and also tried to see it as a graph on 2n vertices where there...

Words such as question do not add information to titles. Please edit the title so that it better describes the specifics of your question. Do not hesitate to make it longer or include a formula if needed. More tips here. (from a bot)Normal Human 21 secs ago
 
4:34 AM
0
Q: How do I solve the following optimization problem

user2475714Hey i was wondering if anyone can help me with the following optimization problem: enter image description here I know the answer to (a) is 100 = 4s + 2(pi)(radius^2) and the answer to (b) is A=s^2 + pi(radius)^2 but what si the answer to (c) and (d)? for (c) do I provide the domain for the are...

 
0
Q: Why are my badges so unhealthy?

JojodmoFor some reason, my badges just lost a lot of weight It's not normal for a healthy badge to be that skinny. I'm getting a little bit worried - what have the SE staff done to make my badges so unhealthy? If that made no sense at all to you, the badges don't seem to have as much spacing as ...

 
0
Q: Using FLT for n=3

NYNUsing FLT for exponent 3, I need to show that if n positive integer is divisible by 3, then there are no x,y,z positive integers such that x^n+y^n=z^. This is what I did: if 3/n then n=3.k, for some k positive integer. Then if (x^3, y^3,z^3) is a solution for the exponent k (x^3)^k+(y^3)^k=(z...

This site uses MathJax formatting of formulas. More tips here. (autocomment)Normal Human 21 secs ago
-1
Q: In S5, we have aba^-1=b^2, b=(12345), find a.

ycex34In S5, we have aba^-1=b^2, b=(12345), find a. I have tried different ways to substitute/rearrange, but none of them worked.

0
Q: Finding the basis of a vector space: question about methodology

JSmithProblem: Find a basis of the vector space $\{x \in \mathbb F^n \mid x_1 + \cdots + x_n = 0\}$. Solution: The vector space in question is exactly $$\left \{ \begin{pmatrix} -x_2 - \cdots - x_n \\ x_2 \\ \vdots \\ x_n \end{pmatrix} \; : \; x_2, \ldots, x_n \in \mathbb F \right \}.$$ Setting one ...

0
Q: Show the preference is non-monotone

anotherdayGiven that the utility function is $u(s_1,s_2)=s_1-a(s_1-s_2)^2$, where $a>0$. How can I show that this preference is non-monotone?

0
Q: Informal axiomatic presentation of elementary logic

Randy RandersonCan someone recommend a book which gives an informal but axiomatic presentation of elementary logic? More precisely, I envision a text with the following features: Takes grammatical sentences (which may be true, false, undecidable, contradictory, etc.) and logical symbols (connectives, quanti...

0
Q: How can you find the domain and range of a rational function?

Sukrit GaneshI was wondering how exactly to find the domain and range of a rational function. I have a test next week and I wish to know a simple and straightforward method to find the domain and range. You can use the function (x+3)(x+4)/(x-2)(x-6) And also (x+3)(x+4)(x+8)/(x-3)(x-2)

Welcome to Math.SE, Sukrit Ganesh. This site uses MathJax formatting of formulas. More tips here. (autocomment)Normal Human 21 secs ago
0
Q: set theory proof verification help

1233211I know this may be routine and simple, but I'm not great at proofs and was wondering if someone could look at my solution and offer corrections or tips? If A and B are sets, prove that $ A \cap \emptyset = \emptyset. $ My proof: Let $x \in A \cap \emptyset $. Then x $\in A$ and $x \in \empty...

Words such as help are uninformative in titles. Please edit the title so that it better describes the specifics of your question. Do not hesitate to make it longer or include a formula if needed. More tips here. (from a bot)Normal Human 21 secs ago
 
5:14 AM
0
Q: Should I vote for (recommending) deletion on an answer that's just had a post notice put on it?

Nathan TuggyThe mods here use post notices more than many sites, which I think is kinda nice. But unfortunately that means the general guidance on how to work with those is a bit lacking. If an answer has a post notice when it shows up in Low Quality Posts review (or, for 4k users, wherever they might stumbl...

0
Q: Add a link within a question or answer?

neuronetI wrote a very long answer to a question, an answer with multiple sections. I would like to be able to link to those sections rather than send people to the post to fish around for a particular section. In html, this is done by adding anchors to the page. Is there a way that we can use these, or ...

 
0
Q: Let x,y,z be integers with x > y. Then xz > yz. Prove or disprove.

Vicky E. RombergerLet x,y,z be integers with x > y. Then xz > yz. Prove or disprove. If the statement is false, modify the hypothesis so that the edited statement is true.

 
2
Q: Page links on homepage broken

RobOn the homepage, page links seem to be broken (http://stackoverflow.com/) The links are defined in HTML like this: <a href="/" title="go to page 2"> <span class="page-numbers">2</span> </a> However, when using filters (http://stackoverflow.com/questions/tagged/c%23) the links work. <a href="...

 
0
Q: Rank of a FreeGroup in GAP

gxydIn GAP > gap> f:=FreeGroup(2);; > gap> Rank(f); Error, no method found! For debugging hints type ?Recovery from NoMethodFound Error, no 2nd choice method found for `RankPGroup' on 1 arguments called from <function "HANDLE_METHOD_NOT_FOUND">( <arguments> ) called from read-eval loop at line 11 o...

Short title. Short question. Rank of a FreeGroup in GAP
0
Q: Value of the alternating sum of binomial coefficients

hussonfrancoisEvaluate $\sum\limits_{k=0}^n\sum\limits_{l=0}^n\left(-1\right)^{k+l}\binom{n+k-l}{n}\binom{k+l}{n}\binom{n}{k}\binom{n}{l}$

0
Q: Question about the bounded requirement of the simple function of definition of stochastic integration

DanielsenLet $W_t$ be one-dimensional Brownian motion, to calculate $\int_0^tW_sdW_s$ by the definition of stochastic integration, one way is use $W^{(n)}_t=W_{[nt]/n}$ to approximate $\int_0^tW_sdW_s$. My question is the definition of stochastic integration require the simple function to be bounded, but...

0
Q: probability density function of a uniformly distributed variable

user3874530http://imgur.com/VdGv6JH I'm a bit confused on how to solve this. This is my work so far: http://imgur.com/oCdydU7 But the last step I have done is apparently wrong. Any help would be appreciated!

0
Q: values that can be attained by random variables

user299065Can a discrete random variables takes the values $+ \infty$ and $- \infty$ ? Can someone explain to me this with an example?

 
5:41 AM
0
Q: For all x in the natural numbers, there exists y in the natural numbers such that x < y ^ x+y=perfect square

Vicky E. RombergerSuppose a pair of positive integers x, y are called square mates if their sum x+y is a perfect square. (Note, square mates is a made up definition.) Then, for all x in the natural numbers, there exists y in the natural numbers such that x < y ^ x,y are square mates. Prove or disprove. If this sta...

0
Q: Bifurcation diagram

CookieConsider the logistic map $x_{n+1}=rx_n(1-x_n)$, whose bifurcation diagram is shown below for $2.4 < r < 4.0$: I need to find a particular value of $r$ so that "attracting $2^k$ periodic points (a result after $k$ instances of period doubling) accumulate". This was one part of my lab assignment,...

0
Q: Differnce between a function and a sequence

AdnanWhat is the difference between a function and a sequence? I have googled it but I have not found any solid answer anywhere.

Consider adding a tag for a broader subject area to which the question belongs. Some of these tags might fit. (autocomment)Normal Human 28 secs ago
 
1
Q: Should I accept an answer even if the issue hasn't been solved?

asprinCase Scenario Let's say I ask a query: How to do x? The answer says: x cannot be done due to some y restrictions. In this case should I accept it even though my problem hasn't been solved? Wouldn't it mislead** others if they happen to stumble upon the question? ** In can be misle...

 
5:57 AM
0
Q: How do I detect collisions in a parametric equation? (Lissajous curve)?

user3832863Let's say you have a simple parametric equation where x= sin(3t) and y=cos(7t). This is a pretty simple parametric equation that generates a relatively complicated Lissajous figure. You can check it out here pretty easily: www.wolframalpha.com/input/?i=parametric+plot (If I presented the enti...

This site uses MathJax formatting of formulas. More tips here. (autocomment)Normal Human 20 secs ago
0
Q: Limits Properties of exponential function

Raheem NajibRudin in his PMA defines: $$E(x)=\sum \limits_{n=0}^{\infty}\dfrac{z^n}{n!} \quad (z\in \mathbb{C}) \qquad (1)$$ He write that "these limits follows directly from (1)" 1) $E(x)\to +\infty$ as $x\to +\infty$. 2) $\dfrac{E(h)-1}{h}\to 1$ as $h\to 0$. I can't understand how the directly follows f...

0
Q: Proofs with for all statements including uniqueness and divides

vroLet $\mathcal{A}$ be a nonempty finite set of positive integers, with $\forall$ r $\in$ $\mathcal{A}$, $\forall$ s $\in$ $\mathcal{A}$ : r|s or s|r. (i). Prove $\exists$t $\in$ $\mathcal{A}$: t|a, $\forall$ a $\in$ $\mathcal{A}$ (ii). Prove t is unique (iii). Suppose $\mathcal{A}$ $\subseteq$ ...

 
1
Q: Page buttons don't work on stackoverflow.com/

JojodmoThe "change page" buttons at the bottom of the website do not work - they simply bring you to the top of the page, but do not change the page. The next button does not work, either. It just brings the user to the top of the page This only happens on https://stackoverflow.com/ It does not hap...

 
1
Q: Badges as motivation

Diego CoutoThere is badges for some kinds of things. I used to only vote an answer up when I face a problem that the answer helped me. But some time later, I saw that some badges could be achieved by numbers of votes during a day and I started looking for questions in which I can vote answers up. I imagine ...

 
0
Q: If $A$ is $G$ - invariant then so is $\bar A$

RiseLet $G$ be a topological group acting continuously on a topological space $X$. Let $A$ be a $G$ - invariant subspace of $X$. Then is it true that $\bar A$ is also $G$ - invariant? (where $\bar A$ is the closure of $A$ in $X$) My attempt - Let $g\in G$ and $x\in \bar A$. To prove that $g\cdot x ...

0
Q: Why is x^2 - 2x > 0 the same as x<0 or x>2

RoyWhy is "x^2 - 2x > 0" the same as x<0 or x>2 and "x^2 - 2x < 0" same as 0

 
6:36 AM
0
Q: Questions with [repeat] are repeatedly tagged with [repeating]

Bhargav RaoPuns Apart, I asked a question How to gather support for tag synonyms?. The answer was to ask on meta to popularize the request. repeat - 1071 Questions "repeat" refers to the action of doing something over again. repeating - 314 Questions (Pi, Pi everywhere) I propose to make repeat...

 
0
Q: Floor function and convergence of the sequence

hussonfrancoisSequence $\{a(n)\}$ of real numbers is such that $\forall\space\lambda\in(1,2)$ sequence $a(\lfloor{\lambda}^n\rfloor)$ has a finite limit. Does it follow that $\{a(n)\}$ is convergent?

0
Q: If $\mathbb_{R}$

DManI am presented with the following problem: Let $T: R^3 \rightarrow R^3$ be linear and define it as such: $T(x) = Ax$ for some $3x3$ matrix $A$. Let $m_1, m_2, m_3 \in R^3$ be linearly independent, call this basis "$m$". Find $[T]^m _m$. In general terms how can I find the matrix of transforma...

Short title. If $\mathbb_{R}$
0
Q: What means size of an edge?

Omid EbrahimiIn page-13 of Graph minors. X. Obstructions to tree-decomposition, $\gamma(G)$ introduced as maximum size of an edge. What is the definition of size of an edge? I think it may be number of edges that incident on terminals of an edge.

Short title. Short question. What means size of an edge?
0
Q: Maclaurin's Series for $sec(x)$ with help of Maclaurin's Series for $tan(x)$

MathematicsIs there any way to derive Maclaurin's Series for $sec(x)$ with the help of Maclaurin's Series for $tan(x)$? Maclaurin's Series for $tan()$ is: $tan(x)=x+\frac{x^3}{3}+\frac{2x^5}{15}+....$

0
Q: The Tower of Hanoi- a mathematical game, proof needed

vroThe Tower of Hanoi is a famous math puzzle consisting of a board with three dowels and a collection of $n$ donuts of different sizes (radii). The donuts are on the first dowel arranged in size order with largest on the bottom and smallest on top. The object of the game is to move all of the donut...

 
7:27 AM
0
Q: Solving for n. Logarithm

user284982How come: 3^n/2^(2n-1) < 0.5 * 10^(-5) will be equal to n = 45 ? Thanks in advance!

Short title. Short question. Solving for n. Logarithm
0
Q: fractional representations of repeating decimal

vroLet $d_1$, $d_2$, $d_3$ $\in$ {0,1,2, $\dots$ 9}. What is the fraction representation of 0.$d_1$$\overline{d_2 d_3}$ ? Prove this in general.

Short question. Tagged proof-writing. fractional representations of repeating decimal
0
Q: How to determine if two 6x6 nilpotent matrices are similar?

DManLet A and B be two $6x6$ nilpotent matrices. How would you prove they are similar?

0
Q: How to find the PDF for Y=e$^x$

Marissa FarinaHow do I find the PDF for $Y$ = e$^X$ when $X$ is $N$(μ,σ$^2$) I have seen the problem where $X$ is $N$(0,1), but I am curious on how to find it given just these parameters?

 
7:43 AM
1
Q: Review audits, banned again - bad luck?

CodeiSirI really didn't want to complain, but I got banned pretty much instand again because of the review audit system. Last week I got banned for seven days, for a audit that I really disagree with. I was mad, as I already had an unclear audit just before. Also I was reviewing hundreds of tasks in the...

 
8:07 AM
0
Q: Can we merge similar question on SO

saurabh kambleWhile searching found similar question some of them are marked duplicated but can we merge such type of question which are left unanswered. Question:(Unanswered Question) Question 1 Question 2 Question:(Answered Question) Answered Question Answered Question How can we achieve to stop dup...

 
0
Q: Unit Plus Nilpotent Is Unit

JosuéI recently had an algebra exam and one of its questions was: Let $R$ be a ring with a $1$ and let $x\in R$ be nilpotent, i.e., there exists a positive integer $n$ such that $x^n=0$. Then $1+x$ is a unit. This is what I did: Observe that $$\begin{align*}\left(1+x\right)\left(1-x\right)&...

 
8:23 AM
0
Q: solving weather problems

user3610659In Quantville the weather follows a peculiar pattern described in the table below: Tomorrow's Weather Sunny Cloudy Windy Rainy Today's Weather Sunny 40% 40% 20% 0% Cloudy 20% 20% 30% 30% ...

Short title. Title contains problem. solving weather problems
0
Q: limsup of a probability

SunkenSI've encountered the following questions: Suppose $X_n \rightarrow X$ in distribution, and $a<b$ Prove that $$ P( a \le X\le b) \ge \limsup_{n\to \infty}P(a\le X_n\le b) $$ I know how to find the limsup of a r.v, but i'm confused on the concept of limsup of a probability. My idea of approac...

0
Q: What is the probability that you have a Straight Flush if you have a Flush?

user123I dont have poker game knowledge. Any suggestion for this? The probability of drawing a flush is .001980439. The probability of drawing a straight flush is 0.00001544.

0
Q: Endpoints of interval where a functions increasing or decreasing

yavuzwhen determining the intervals in which the graph of a function increase or decrease, Some boks include the ends while others do not. which one is true or which one is more prefreable?

0
Q: inclusion-exclusion principle proof (Without summations)

vroSuppose $\mathcal{A}$, $\mathcal{B}$, and $\mathcal{C}$ are sets with $\mathcal{A} \cap \mathcal{B} \cap \mathcal{C} = \emptyset$. Then $| \mathcal{A} \cup \mathcal{B} \cup \mathcal{C} |$ = $|\mathcal{A}|$ + $|\mathcal{B}|$ + $|\mathcal{C}|$ Prove or disprove. If false, modify statement to make i...

Tag (proof-writing) should not be the only tag a question has. Please add a tag for a subject area to which the question belongs. (autocomment)Normal Human 21 secs ago
0
Q: Programming and ZFC

WakakaSuppose I have a simple program that implements an algorithm (say depth-first search), written in a simple imperative programming language with the standard for loops, recursions, conditional statements and so on. It takes in a well-specified input and has a well-specified output. Suppose I want...

Short title. Tagged proof-theory. Programming and ZFC
 
8:43 AM
0
Q: Allow hiding of individual job listings

Brad WerthI've been checking out the Careers Unificintegration and so far it seems quite nice. One problem, however, is that there is no way to remove jobs that have been previously evaluated to be a poor fit. Ideally, there would be a "Not Interested" button, or something, that would cause the job to be h...

 
0
Q: Inverse Laplace Transform (Natural Logarithm Case)

Maximilian HuangI have a problem about Inverse Laplace Transform, I would be appreciated to get your help for solving this problem (It took me about several hours to think but didn't come up with any solution), please find the inverse laplace transform of : Problem Here where "a" is an constant.

 
0
Q: How often is SEDE updated?

Julian E.I am currently trying to go for the Unsung Hero badge in order to get a new hat on the Winter Bash 2015. Now, to the actual question. I have asked in the past on how to find how many zero-score accepted answers I have and got linked to two resources. The one being to perform a simple search as ...

 
0
Q: Initial Value Problem of Difrrential equation

kalpeshmpopatLet $y(x)$ be continuous solution of initial value problem $\frac{dy}{dx}+2y=f(x),y(0)=0$. Where $f(x) = \left\{ \begin{array}{rcl} 1 & \mbox{for} & 0 \leq x\leq 1 \\ 0 & \mbox{for} & x>1 \end{array}\right.$ Then FInd $y(\frac{3}{2})$ Answer is $\frac{sinh(1)}{e^2}$ I have tried the follo...

0
Q: Conjugacy classes of a group

kalpeshmpopatWhat is number of conjugacy classes in the permutation group $S_{6}$ I only knows that $o(S_6)=6!$

Short title. Short question. Conjugacy classes of a group
 
9:19 AM
posted on December 16, 2015 by Oceinic

На enSO давно есть возможность возможность помечать сообщения общими, но она практически не используется. Зато на ruSO этот формат на данный момент является одним из самых обсуждаемых. В этом ...

3
Q: Self deletes shouldn't validate spam flags

FlexoQuite a few users are prone to flagging as spam any/all terrible questions, simply because they're terrible. This is incorrect use of the spam flag and if a moderator catches it the flags will be declined. Unfortunately if the question gets deleted, including by the post owner, the flag gets di...

 
9:36 AM
0
Q: Showing that $H$ is normal

user96343This question has been bugging me for a while. Suppose that $H$ is a subgroup of $G$ such that $Ha\not=Hb$ implies that $aH\not =bH$. I need to show that $gHg^{-1}\subset H$. Now, take $x\in gHg^{-1}$. Suppose it is not in $H$. Then $x=ghg^{-1}$ for some $h$. As $x\in H$, $Hx\not=H$ whih implies...

 
9:53 AM
0
Q: Can you simplify this term?

emcor$$X=\frac{\left(\frac{c}{r^2}+\frac{1-c}{(1+r)^{T+1}}\right)}{\left(\frac{c}{r}+\frac{1-c}{(1+r)^T}-1\right)}$$

Short title. Short question. Can you simplify this term?
 
-2
Q: This winter is weak!

Ilya_GazmanI remember getting much more hats last year... I been getting hats for editing, commenting or just staring in the screen. And now... I barely can see people with hats, what do you think about it?

0
Q: Diagonal lines in table cell on mathjax

Nehorai Tables of numbers Is it possible to put a "x\y" style legend in one cell (maybe be made up of several cells) of a table? It would be great to make the 'x' slightly lower while the 'y' slight higher. Please see the picture below for illustration: Thank you very much.

 
0
Q: $x^4 = -1$ (mod $p$) implies p = 1 mod 8

LudwwwigLet $p$ be an odd prime. Show that $x^4 = -1$ (mod $p$) $ \Leftrightarrow p = 1$ (mod $8$)

0
Q: Is my proof of $C_G(H) \le N_G(H)$ correct?

user264885Let $x\in C_G(H)$. This means $xh = hx$ for $h \in H$. Then $xH = Hx$ (This is that part I'm not so sure about). Hence, $x \in N_G(H)$, so that we have $C_G(H) \le N_G(H)$.

0
Q: Can someone give me points to solve this question?(Sobolev Spaces)

ShalalehDetermine whether the first and second order weak derivatives of the following function $f: \Omega \to \Bbb R $ exist, and if exist what they are.\begin{align} \end{align} $a. \Omega_1 =(0,2\pi), f_1(x):=|\cos x|. $\begin{align} \end{align} $b$.Determine whether the function is contained in the ...

0
Q: Hilbert's tenth problem

Mary StarAre the folowing equivalent? Hilbert's tenth problem and the theorem of Matiyasevich, Davis, Putnam and Robinson that the diophantine sets are equial to the enumerable sets?

Short title. Title contains problem. Short question. Hilbert's tenth problem
 
10:26 AM
0
Q: Compact operator for which the image of the closed unit ball is not compact

mathcounterexamples.netDo you have an example of a compact operator for which the image of the closed unit ball is not compact?

 
1
Q: Community user wearing hats for voting?

AniketI just checked on the profile of the Community user and saw that its avatar was wearing hats. Fine, he should be allowed to do so. But I checked that he was having 2 hats for voting on questions and answers, one for voting on apps and the other for 250 votes in 7 days. But the Community user's pr...

0
Q: Tags in "Fill out your CV" are borked

Sumurai8The fields for adding tags in the "Fill out your CV" dialog on the new integrated jobs site seem borked. The tags are displayed too far down. If a tag in the middle is deleted, all tags to the right disappear.... until you press the right arrow key. I assume an extra newline is added, which wil...

0
Q: I just got a tag badge for a for a language I don't even know

SuragchI got a bronze tag badge today for Objective-C even though I've never written a single Obj-C program in my life. I can guess my way through the meaning sometimes, but I certainly can't produce the syntax by myself. The reason I got this badge is that I have added Swift version answers to old Obj...

 
10:47 AM
0
Q: Weyl group of $V_{4}$ in $S_{4}?$

LonrAbstractCan anyone please explain what a Weyl group is and what a Weyl group of $V_{4}$ in $S_{4}$ is? I do not understand this exercise.

Short question. Question contains please. Weyl group of $V_{4}$ in $S_{4}?$
0
Q: another question of solveb space

ShalalehLet $\Omega=(-1,1) *(-1,1). f,g: \Omega \to \Bbb R$ are defined by $$ f(x,y) = \begin{cases} 1, & \text{$x\le 0$} \\ 0, & \text{elsewhere} \end{cases} $$ $$g(x,y)=||x-y||$$ Determine whether $\frac{\partial f}{\partial x},\frac{\partial f}{\partial y},\frac{\partial g}{\partial x}$ and $\frac{...

Words such as question do not add information to titles. Please edit the title so that it better describes the specifics of your question. Do not hesitate to make it longer or include a formula if needed. More tips here. (from a bot)Normal Human 21 secs ago
 
11:03 AM
0
Q: Express the followin equ as a single log

NookjayI'd like help with 2 + 3Logx. With detailed workings on the question. Thanks.

0
Q: Number Based sign series puzzle

RAJEEV SHRIVASTAVAGood Morning! This is Rajeev, How are you? I hope you will be fine. I am gain sending you a numeric based series puzzle for a hope of solution. If possible, please help to solve. Importance High!!!!!!!!!!!!!!!!! Dear Sir / Madam,   I am putting a signs series in front of you which based on rea...

This site uses MathJax formatting of formulas. More tips here. (autocomment)Normal Human 21 secs ago
 
2
Q: I didn't get the bounty for a question i answered

CyrilThis question was on bounty Multiple partners in a family tree in d3.js? Yesterday it was running on grace period of 24 hours, and on expiry i didn't get any bounty. Now the history of the above question is that the person had accepted my answer before starting the question on bounty. In the p...

-1
Q: Should there be a css-only tag?

gpinkasAfter seeing this question as an example for some others I have seen in the past, wouldn't it be convenient to have a css-only tag to emphasize that the poster does not want a Javascript/jQuery answer? It seems inevitable that these come up, even if it's stated in the question or comments that a ...

 
0
Q: Confusing solution to the limit of an implicit function?

Jack$$\frac{8}{3}=\frac{\log{x}}{\log{y}}-\frac{\log{y}}{\log{x}}$$ When I graphed this implicit function on desmos (https://www.desmos.com/) it appeared as if there were two solutions as $x\to{0}$ from the positive direction: $y\to\infty$ and $y\to{0}$. However, neither of these solutions make much...

Title contains confus. Question contains please, please. Confusing solution to the limit of an implicit function?
0
Q: simulation for post office queuing problems

user3610659Simulate the following situation. Attach the code as part of your submission. At a post office, customers enter a single line waiting to be served by any one of two clerks. Every minute there is a 60% chance that a new customer arrives. If there is no one in line and a server is free, the cus...

Consider adding a tag for a broader subject area to which the question belongs. Some of these tags might fit. (from a bot)Normal Human 23 secs ago
0
Q: Are these the same ?

MohammedIs x•y=60 same as y=60-x If not then y is equal to what ( in x•y=60 )? I'm confused .. I'm not sure what should y be because it multiplied ... ------------/----------/-----------/---------- This is just a copy , cannot meet .... Is x•y=60 same as y=60-x If not then y is equal to what ( in ...

Short title. Are these the same ?
0
Q: Unitary Operator on Hilberspace to show that Fourierbasis is a maximal Orthogonal Set

spookyI have looked at the proof Proving that the Fourier Basis is complete for C(R/$2*\pi$ , C) with $L^2$ norm but am having trouble understanding the argumentation about the Hilberspace. I think the proof is quite nice and would like to understand it, and also need to refresh my Functional analysis....

Welcome to Math.SE, spooky. This site uses MathJax formatting of formulas. More tips here. (from a bot)Normal Human 21 secs ago
0
Q: Propositional formulas 3

p.mcwhat are the difference between: A → (B → C), (A → B) → C, A∧B → C, A → B ∧C, (A → B)∧(B → C)? Why we can not write A → B → C And the same questions about: A ↔(B ↔ C), (A ↔ B) ↔ C, A∧B ↔ C, A ↔ B ∧C, (A ↔B)∧(B ↔ C)? Why we can not write A ↔ B ↔ C? Can someone explain me this ?

Short title. Title ends with a digit. Propositional formulas 3
0
Q: The diagonals of hat matrix in linear regression

user5680412I read from a book that for linear regression problem with intercept: $$Y = X\beta + e, e\sim N(0,\sigma^2 I)$$ The diagonal entries of the hat matrix $$H = I - X(X'X)^{-1}X'$$ can be expressed by $$h_{ii} = \frac{1}{n}+(x_i-\bar{x_i})'(\mathcal{X}'\mathcal{X})^{-1}(x_i-\bar{x_i})$$ where ...

Questions tend to get more attention when they have a tag for a broad area of mathematics relevant to the question. Some of these tags might fit. (from a bot)Normal Human 21 secs ago
0
Q: A finite group containing only elements of order 1 or 2 must have even order

hbgSuppose G is a finite group in which every element has order 2 or 1, and that G contains some element different from the identity element. How can I explain that the order of G must be even?

 
11:40 AM
0
Q: What license are the hats puplished on?

ZaibisI really love the way my actuall hat is improving my avatar. Would I be allowed to make a copy of that hat and edit it in my avatar after the event is over?

 
0
Q: Can't solve this it's confusing word problem

MohammedThe space of a rectangle is 60 , If you add 3 to the length of the rectangle and 1 to the width then the new space of the rectangle is 90. What is the length and width of the original rectangle ? I tried doing this X is the length Y is the width X*Y=60 (X+3)(Y+1)=90... But it didn't work f...

Title contains confus, problem. Question contains please. Can't solve this it's confusing word problem
0
Q: Deriving Quadratic Function Help

None of YerBizHow will you derive the quadratic function if you only have the zeros which are 2+ √7 and 2-√7?

Welcome to Math.SE, None of YerBiz. Words such as help are uninformative in titles. Please edit the title so that it better describes the specifics of your question. Do not hesitate to make it longer or include a formula if needed. More tips here. (autocomment)Normal Human 21 secs ago
0
Q: Do you know the formula?

user299126I have three data: coordinates (x,y) angle (degree) distance (meters) How should I calculate new (x,y) ? for example with these data : begining coord. (100,100) , angle - 30 , walked 2meters. What is the result? Sorry for formulation of this topic and explanation of my problem, but i really ...

0
Q: Is there any classifier which is able to make decisions very fast?

DavidMost classification algorithms are developed to improve the training speed. However, is there any classifier or algorithm focusing on the decision making speed? I can achieve enough training data,and endure the long training time.

 
12:25 PM
0
Q: About $f(x)= f(\frac{1}{x})$

mickConsider the equation $$f(x)=f(\frac{1}{x})$$ Where we want $f$ to be real-meromorphic. Are all solutions $f$ of the form $$f(x) = g({x}{1+x^2})$$ Where $g$ is a continu function ?

 
12:42 PM
0
Q: Make [openerp] a synonym for [odoo]

Ludwik TrammerIn 2014 OpenERP was renamed to Odoo. Currently in StackOverflow Odoo is a synonym of OpenERP, but it should be the other way around. The old name should redirect to the new one.

 
0
Q: Non-Constant Acceleration

user2704821How do we calculate time that we need for specific distance if we have non-constant acceleration. I have some formulas for other variable but don't have for time: Speed at time: $$ v(t) = v_0+at+\frac{j(t^2)}{3} $$ Distance at time: $$ x(t)=x_0+v_0t+\frac{a(t^2)}{2}+\frac{j(t^3)}{6} $$ Acceler...

 
0
Q: Bug in count of votes of the user

IdenticonI just signed in my account of stack overflow, and the next badge I had chosen was Civic Duty. In that, it showed that I had 164 votes cast, but in the right side, it showed I had 161.

3
Q: What happens to up and down votes before you have the privilege to cast such votes?

hkBstAt 1 reputation you have neither the privilege of upvoting nor the privilege of downvoting. If you try it anyway, you get some message from the system, from which I understood that your vote is saved until the moment you get the relevant voting privilege and then (retro-actively?) cast. Perhaps I...

 
1:15 PM
-3
Q: How do I get date-based hats after the day is over?

harshad pansuriyaYou receive the Sufganiyot hat for asking, answering, or voting on December 14th. I missed that day however. So how I can earn that hat now?

0
Q: Bug New Nav: Home Tab Does not Allow Change in Items per page

JRSoftyWhen attempting to adjust the number of items per page on the Home tab I am only brought to the top of the page. No change is made.

0
Q: Job types other than Permanant and Contract

ViniIn the search bar there is an Option to filter Job type and listing two Options apart from any. So if the job doesn't fall in one of the two categories what does it fall into? When a job is posted, the job type isn't a necessary field? When i filter the results based on job type i get 0 resul...

 
0
Q: Evolution of Hysteresis

BahaI have been trying to figure out an expression for evolution of a hysteresis shown in the picture below. It is given that both A and d are functions of 'phi'. The shape of the geometry is a perfectly symmetric parallelogram. There is an expression used to define evolution of A with respect to a...

0
Q: Algebra review for Spivak Calculus

ayNONEI got a bit bored with High School maths so I picked up a copy of Calculus by Spivak. I am really enjoying the book and have found that the proofs and theorems aren't as hard as others have made them out to be. However, I am struggling a bit with the algebra. The education I have received thus fa...

Please don't use (self-learning) tag just because you were self-studying. This tag is only for questions about the process of self-studying. (from a bot)Normal Human 21 secs ago
0
Q: Why is $\sin(\arccos(y)) = \sqrt{1-y^2}$?

LisaWhy is: $\sin(\arccos(y)) = \sqrt{1-y^2}$ ? I thought maybe transforming sin to cos: $\cos(\frac{\pi}{2} - \arccos(y))$ but it doesn't get me anywhere. Hint?

0
Q: Prove that $[a,b]$ is compact

luka5z Let $a<b$ be real numbers. Prove that $[a,b]$ is compact. Below I present my solution. I thought it's good enough, but my TA said it's incorrect. I don't see where there is a problem. Could you help me find it? My proof: Let $\{U_i\}_{i\in I}$ be an open cover of $[a,b]$. Let $$S=\{x\in [a...

Short title. Tagged proof-verification. [Prove that $[a,b]$ is compact](math.stackexchange.com/q/1578306)
0
Q: Question about the notation of vectors in respect to their domains

Sharan DuggiralaA column vector $a$ is said to an element of R^(4) when $a$ is [1, 2, 3, 4]' (Transpose of this row vector). Therefore my question lies in the fact that I do not understand how row vectors can be represented with this notation. I also understand that Matrices can be represented by for e.g. R^...

 
1:47 PM
0
Q: Specific stochastical/statistical question

TobiasI'm currently programming a small script and have a statistical problem: I have 24 sequences, consisting of 4 different characters and a length of 8 characters. It is necessary to group the sequences to 8 sequences per group. To make it more clear, here are two example groups: Example 1: CAAGTC...

Welcome to Math.SE, Tobias. Words such as question are uninformative in titles. Please edit the title so that it better describes the specifics of your question. Do not hesitate to make it longer or include a formula if needed. More tips here. (from a bot)Normal Human 21 secs ago
0
Q: Digits with Nudges-Combinatorics

PiComedianGiven a $2$-digit sequence (which can start with $0$), let us define a nudge as the operation of either increasing or decreasing one of the digits by $1$, or changing a $0$ to a $9$ or a $9$ to a $0$. For example, we can nudge $19$ to get $09$, $29$, $18$, or $10$. Suppose $S$ is a set of $2$-di...

0
Q: Who can help me proof $l(M)=dim_{K}M$

AngleLet $A$ be a basic finite dimensional algebra over an algebraically closed field $K$, and let $M$ be a finite dimensional right A-module. Show that $l(M)=dim_{K}M$, where $l(M)$ is the length of the composition series of $M$.

Words such as help do not add information to titles. Please edit the title so that it better describes the specifics of your question. Do not hesitate to make it longer or include a formula if needed. More tips here. (from a bot)Normal Human 21 secs ago
 
1
Q: No winterbash-2015 hats visible on profile page

arc_lupusWhile the winterbash-hats are visible on (f.ex.) academia.se, I can't see the snowflake on stackoverflow. Nevertheless according to meta-posts SO should participate. Is that a bug, or something else?

0
Q: On Jobs, how do I sort by most recent matches?

AndyIt appears that I have two options to filters jobs: Matches and Most Recent. If I select "Most Recent", it sort mostly by the newest first. The very first posting is not the most recent, but it is featured, so I assume that's intentional. However, if I search by "Matches", the job posting t...

5
Q: "Edit CV" on every user account (probably moderator only)

Martijn PietersVisiting any user account I see an Edit CV link: The link leads to https://stackoverflow.com/jobs/cv/edit, letting me edit my own CV. This is almost certainly a moderator-only problem. Either only show me the link on my own userpage, or if moderators are supposed to moderate CVs (which I ...

0
Q: How do theoretical physicists work?

Dirk BruereHow do they spend their day? Looking out the window trying to think up something new? Learning new maths? Reading other people's papers? And how is their productivity measured beyond churning out papers?

0
Q: New Tab Bug - New / Custom Tab keeps disappearing

GorblesI've found this thread which seems appropriate but I'd like to ask how tabs are stored. I rarely empty my cache, I rarely delete cookies . . . is this a localStorage thing (been clearing that out recently for testing stuff at work)? This is on Firefox Developer Edition, across my work laptop (Wi...

 
0
Q: Integration of ordinary differential equations

J. DoeI was watching a lesson in youtube on the wave equation and couldn't figure out how the person completed one of the steps where he integrated a second derivative.

 
2:09 PM
0
Q: A relation on a structure defined as a set? Help with an example from Dalen's Logic and structure

AlexanderI'm not able to understand the folowing example from Dirk van Dalen's logic and structure. What is confusing me is the relation defined on the structure and the predicate $P $. We have a set $A=\{0,1 \} $ and a relation defined on $A $ as the set with two elements $\{\langle 0.1 \rangle , \lan...

 
-3
Q: "Links to jsfiddle.net must be accompanied by code." Not correct imo.

Jp HoutenI have stumbled upon this issue many times, when I ask a question accompanied by a JSfiddle I must insert code. This is in my opinion not needed because : The JSFiddle contains code Some "short" coded code's do not need clarification. Code can make the article more confusing. IMO, this should...

 
0
Q: nullity of a matix

TaniLet T be a 4*4 real matrix such that $T^4=0$.Let ki =dim Ker $T^i$ for i=1,2,3,4.Then which of the following is NOT a possibility for the sequence k1,k2,k3 and k4 1)k1 = 3,k2 = 4,k3 = 4 and k4 = 4 2)k1 = 1,k2 = 3,k3 = 4 and k4 = 4 3)k1 = 2,k2 = 4,k3 = 4 and k4 = 4 4)k1 = 2,k2 = 3,k3 = 4 and k...

Short title. nullity of a matix
 
2:32 PM
0
Q: Fresh jobs link needs some sort of tooltip

JonHWhat exactly does this link do? I know it shows additional jobs and the page refreshes but can someone tooltip this?

1
Q: More jobs for company X doesn't work, shows all jobs?

JonHThere is a link on a company page that says: More jobs at company X I would of thought this would filter the jobs for that specific company, but it doesn't. Here's the screen shots: When I click this link I am presented with over 2k jobs at QuickenLoans but when I click any of these jobs th...

0
Q: Can enter key act like tab key when picking tags

JonHWhen I enter a tag on any of the sites if the returned result is one tag and I hit enter shouldn't it act like pressing the tab key? Feature request!! I'd like to be able to hit the enter key when I select tags as long as there is one tag coming up. For instance, If I type jobs in the tag fie...

0
Q: Allow tags to be clickable showing jobs that also have those tags

JonHCompanies post what technologies they use via the tags feature. Feature request, allow the tag to be clickable showing me other jobs / companies who also are tagged with that technology. Benefit of this, potential candidate sees a job he / she is interested in with the tag reporting services. ...

0
Q: "Only able to post once every 90 minutes"

Jp HoutenI am having issues with this option, although I understand the principle that "newbies" may not post many times to prevent spam. At my work we all work from the same ip (like post of us). This is not ideal for us, since we all developers, with 8 people, we sometimes have questions, and when we ...

 
0
Q: how to plot joint pdf of 2 normal distributions

lucifer157I have: $F(\omega|x) \sim N(1, (1+x)^2)$ $G(\sigma|y) \sim N(1, (1+y)^2)$ How can I calculate and plot the joint pdf of $\omega$ and $\sigma$ for $x,y\in [0,1]$?

0
Q: Proof of an inequality with the help of maximal operator

bjk1806I want to prove an inequality such that $$ \int_{B}|f(y)|dy\leq |B|^{1-\frac{1}{p}}\|f\|_{L^p(B)}, $$ where $B\subset\mathbb{R}^n$ is a ball, $p>1$ and $\|f\|_{L^p(B)}=(\int_{B}|f(y)|^pdy)^{\frac{1}{p}}$. This inequality is a direct consequence of Hölder's inequality. But i want to prove this in...

Words such as help do not add information to titles. Please edit the title so that it better describes the specifics of your question. Do not hesitate to make it longer or include a formula if needed. More tips here. (from a bot)Normal Human 21 secs ago
0
Q: Can't solve this equation: ( (1+i)^-n ) / i = -293953 / ( ( (1+i)^-5 ) - 1)

LanderT( (1+i)^-n ) / i = -293953 / ( ( (1+i)^-5 ) - 1) i = 0.03 1.03^-n / 0.03 = -293953 / ((1.03^-5)-1) I end up with this: -n = ln(1.03)/ln( (-293953-0.03) / ((1.03^-5)-1) ) n= -375.9133691 However, I know the answer should be 15. What am I doing wrong?

This site uses MathJax formatting of formulas. More tips here. (autocomment)Normal Human 21 secs ago
0
Q: Question on the sheaf property with either covering sieves or generated covering sieves

jeffreyLet $C$ be a category equipped with a Grothendieck pretopology. Let $U\in C$ be an object. A sieve on $U$ is called covering if it contains a covering family $U_*=\{U_i\to U\}$ from the pretopology. Clearly, there may be many covering sieves on $U$ containing the same covering family $U_*$ and t...

0
Q: How is X/Y distributed when X and Y are uniformly distributed on [0,1]?

AndrewLet $X$,$Y$ be uniformly distributed continuous random variables on [0,1]. How is the random variable $X/Y$ distributed?

Short question. [How is X/Y distributed when X and Y are uniformly distributed on [0,1]?](math.stackexchange.com/q/1578404)
0
Q: Show that a group of order 33 cannot only have elements of order 11

user299162Attempt: Assume $G$ only has elements of order 11 and that there exists $x \in G$ such that $o(x) = 11$. Then $x^{o(G)} = e$. $x^{33} = (x^{11})^3 = e \implies x^{11}$ has order $3$. Contradiction.

0
Q: How to proof mathematical induction? ${(n+1)}^{n+1}>{(n+2)}^n$

YoungI would appreciate if somebody could help me with the following problem Q:How to proof mathematical induction? $${(n+1)}^{n+1}>{(n+2)}^n$$

0
Q: evaluate $lim _{x\to \infty} (3x^2-x^3)^{\frac{1}{3}}+x$

gbox $lim _{x\to \infty} (3x^2-x^3)^{\frac{1}{3}}+x$ can I look at $lim _{x\to \infty} (3^{\frac{1}{3}}x^{\frac{2}{3}}-x+x)$?

 
2:50 PM
0
Q: checkbox control with no action buttons on job messages

JonHThe tab messages shows you specific messages (incoming / outgoing) within the jobs tab of stackoverflow. But the checkboxes do not have any action buttons associated with them, so I don't see any purpose in having them: So I can check the checkbox but it doesn't really mean anything.

-2
Q: Rotate hats in 3D space

Wouter HuysentruitThis is a feature request. Currently I'm facing this problem: As you can see, I cannot use it without chaning my profile image. It would be great if we had 3D rotatable hats next year.

 
3:02 PM
0
Q: Launchpad account recognized on some stackexchanges sites, not all

Stéphane GourichonContext My StackExchange account lists in "My Logins" several providers, including Launchpad. There is written: Log in or sign up on any Stack Exchange site using these accounts Expected When not logged in, I can use launchpad login on any stackexchange sites. When logged in via askubun...

1
Q: Search needs to be updated to work for jobs

JonHBringing jobs as a new tab on the site in general may bring confusion in regards to the global search box at the top. It either needs to work for a job now or we need to explicitly state that the search does not work for jobs. Newcomers or even people who have been here may do this: To me th...

 
0
Q: One-to-one to prove onto

brandaoI stumbled across a problem where I had to prove that a given function is one-to-one and onto. Curiously, I found that in order to prove that the function was onto I had first to show that it was one-to-one. Is there an instance of a proof that a given function is onto that needs the function to ...

 
1
Q: Distance checkbox controls lose value after search companies link is clicked

JonHSteps to reproduce... Click search companies Hover over location and click the allows remote / offers relocation / offers visa sponsorship checkboxes (or click all three of them) Pick any job Click the search companies tab yet again Filter is gone, I was hoping it would stay.

0
Q: Back button takes me back to page 1 when that was not the previous page

pixelmeowI've tried on Meta.SO and SO. If I'm on questions page 2 and I go into a question, then hit the back button, I should go back to questions page 2 (or 3 or whatever). But it's taking me back to questions page 1. This did not happen on Fantasy&SciFi.SE. Is this a bug or is it just my browser? I'm ...

 
0
Q: For which $n_o \in \mathbb{N}$ is it possible to show with induction that $2\log_2(n) \leq n$ applies?

fragantFor which $n_o \in \mathbb{N}$ is it possible to show with induction that $2\log_2(n) \leq n$ applies? for all $n \in \mathbb{N} $ with $ n \geq n_0$? How to proceed to such questions? Hope somebody can help

0
Q: priority between set subtraction and set union

Alejandro Marcos AragonIf I write $ \Omega \setminus \Gamma \cup \Theta$ this means $ \Omega \setminus \left( \Gamma \cup \Theta \right) $ or $ \left( \Omega \setminus \Gamma \right) \cup \Theta$?

0
Q: N - K = R : N , K coprimes.

Jair ReyesGiven N and R we know that N, R are coprime numbers. So.. How can I proove that N - K = R: R will be coprime.

Short title. Short question. N - K = R : N , K coprimes.
0
Q: Solution about a Michael Aschbacher exercise

FedericoI really need the solution of the exercise 11.4 (page 174) in M. Aschbacher, Finite group theory. Here is the text of the exercise. Let $G$ be a finite group, $p$ a prime, $\Omega$ a $G$-invariant collection of $p$-subgroups of $G$, $P$ a $p$-subgroup of $G$, and $\Delta\subseteq P\cap\Omega...

 
3:20 PM
0
Q: Appeal Hold on Question

Seth KitchenI'd like to appeal What video streaming/audio streaming is used in Microsoft HoloLens? This question: how to appeal a closed question? says I should link it here and explain my case. While my question may or may not be suited for Electrical Engineering Stack Exchange (it was being debated and f...

0
Q: What exactly is most recent given days are ranked lower

JonHOn the jobs listing page if I sort by most recent for a specific company then I was hoping the results would show up based on most recent number of days, followed by weeks, months, etc. But this appears to be pretty random:

 
0
Q: Exercise from Implicit Function Theorem's chapter

B. RivasLet $f:U \subset \mathbb{R}^2 \to \mathbb{R}$ be a continuous function such that $(x^2+y^4)f(x,y)+f(x,y)^3=1$ for any $(x,y) \in U$. Prove that $f \in \mathcal{C}^{\infty}$.

Title contains exercise. Short question. Exercise from Implicit Function Theorem's chapter
0
Q: Prove that $\int_a^b \left( \int_c^d f(x,y)dy\right) dx=\int_c^d \left( \int_a^b f(x,y)dx\right) dy$

luka5z Let $f$ be continuous function on $[a,b]\times [c,d]$. Prove that Prove that $$\int_a^b \left( \int_c^d f(x,y)dy\right) dx=\int_c^d \left( \int_a^b f(x,y)dx\right) dy$$ First of all, note that $ \int_c^d f(x,y)dy$ is continuous in $x$ and $ \int_a^b f(x,y)dx$ is continuous in $y$ so bo...

0
Q: Can you help me to understand a proof about equivalency of polynomials?

J.DoeThe lemma says that if m and n are natural numbers and m not divisible by n and $\zeta_m^j \equiv \zeta_m^k \pmod n$, then $\zeta_m^j = \zeta_m^k$ $\zeta_m$ is here a primitive mth root of unity of the complex numbers so $e^{\frac{2 * \pi * i}{m}}$. Now the proof is the following: We multiply t...

Title contains help. Tagged proof-explanation. Can you help me to understand a proof about equivalency of polynomials?
0
Q: $x^a\equiv 1\pmod{p} \Rightarrow x \equiv 1\pmod{p}$

AnnoyingEpsilonWe know: $\gcd(a,\phi(n)=1$ and $a,n,x>0$. Show that $\gcd(x,n)=1$ and $x^a\equiv 1\pmod{p} \Rightarrow x \equiv 1\pmod{p}$ My Attempt: Using Euler's Theorem I know that: $x^{\phi(n)}\equiv 1\pmod{p}$ where $\gcd(x,n)=1$, is this enough to prove that $\gcd(x,n)=1$? Deducing Since $x^a\equiv ...

A title should not be all-MathJax; having some plain text helps with search and navigation. (from a bot)Normal Human 20 secs ago
0
Q: Prove that if [math]7^n-3^n[/math] is divisible by n>1, then n must be even.

Deepak GuptaI tried using factorization of a^n-b^n for odd n in an attempt to work through to a situation where the factors are such that they cannot have n as a factor. But I reached nowhere. Here's how I proceeded - [math]a^n-b^n=(a-b)\left(a^{n-1}+a^{n-2}b+a^{n-3}b^2+\dots+a^2b^{n-3}+ab^{n-2}+b^{n-1}\rig...

0
Q: Is there a non-trivial cyclic quotient group of a non-cyclic group?

ZudwaLet G be a non-cyclic group and H be an non-trivial normal subgroup. Can we state that G/H is non-cyclic?

This site uses MathJax formatting of formulas. More tips here. (autocomment)Normal Human 21 secs ago
0
Q: How to proof an equation using additions theoreme?

RayofCommandI am stuck at one task, it's to proof the following equation using addition theorems. From a draft I understood that the equation is correct or true. But that's it. What's a good way to explain it mathematically? $$sin (x)+ sin(y) = 2 sin (\frac{x+y}{2}) cos(\frac{x-y}{2})$$ As usual, any help ...

Consider replacing (analysis) with a more specific tag for the relevant branch of analysis. (autocomment)Normal Human 20 secs ago
 
3:35 PM
0
Q: Migration of questions loses critical information

JonHThis question was viewed close to 800 times Apply pop-up does not make it clear which version of resume will be attached. Suddenly this question was migrated from Meta Stack Exchange to Meta Stack Overflow. Couple of things here: When migrated a question the information about the migration ...

 
0
Q: If $q, r \in \mathbb{R}, x \in \mathbb{R}^+$ then (x^q)^r=x^{qr}

lorenzoI'm stuck on this exercise from Tao's Analysis 1 textbook: show that if $q, r \in \mathbb{R}, x \in \mathbb{R}^+$ then $(x^q)^r=x^{qr}$. DEF. (Exponentiation to a real exponent): Let $x>0$ be real, and let $\alpha$ be a real number. We define the quantity $x^\alpha$ by the formula $x^\alpha=\li...

 
3:53 PM
1
Q: Distance dropdown disappearing

AHigginsThe distance dropdown in the Location box will not let me select a value. Steps to reproduce: Search for anything Pull up the Location --> Distance menu Hover over any value in the list The entire dropdown disappears. If I click the Distance menu, then use my keyboard to navigate and sele...

 
00:00 - 16:0016:00 - 00:00

« first day (44 days earlier)      last day (530 days later) »